LSAT and Law School Admissions Forum

Get expert LSAT preparation and law school admissions advice from PowerScore Test Preparation.

 Administrator
PowerScore Staff
  • PowerScore Staff
  • Posts: 8916
  • Joined: Feb 02, 2011
|
#40713
Complete Question Explanation
(The complete setup for this game can be found here: lsat/viewtopic.php?t=3641)

The correct answer choice is (A)

If Y is fourth, then from the third rule S must be first or third (S can never be second as determined during the setup). And, if S is first, then Q must be third (from the second rule), and if S is third, Q must be first (as determined during the setup), meaning Q and S rotate between the first and third articles. Both are nutrition articles, which results in the deduction that a nutrition article is first and third, and from the first rule a finance article must be second:

PT68_Game_#4_#18_diagram 1.png
The remaining articles to be placed are J, G, R, and H, and of course from the fourth rule J :longline: G :longline: R. So, at this point, the placement of the articles is limited. We’ll address that in a moment.

Because of the first rule, the last three spots are also restricted. Two must be finance, and one must be nutrition (because the first four articles already consist of two nutrition articles, one finance article, and the one wildlife article). Thus, the two remaining finance articles must be fifth and seventh, and the one remaining nutrition article must be sixth:

PT68_Game_#4_#18_diagram 2.png
With this information in hand (and also because of the original setup Not Laws), we can determine that J cannot be among the last three articles edited, and thus J must be edited second (which determines that answer choice (A) is the correct answer.

To complete the diagram for academic purposes, R is the sole remaining nutrition article and must therefore be sixth. From the fourth rule G must be fifth, leaving the random, H, for seventh:

PT68_Game_#4_#18_diagram 3.png
As mentioned above, J must be second and answer choice (A) is the correct answer.
Overall, this is a very tough question, and it really reveals how intertwined the rules are in this game.
You do not have the required permissions to view the files attached to this post.

Get the most out of your LSAT Prep Plus subscription.

Analyze and track your performance with our Testing and Analytics Package.